Convolution of a Gaussian with itself from the definition

Click For Summary
The discussion focuses on finding the convolution of the function g(x) = e^{-πx²} with itself using the definition of convolution, specifically avoiding the Fourier Transform. Participants express difficulty in applying integration by parts, often concluding incorrectly that the convolution equals zero. A suggestion is made to complete the square in the integration process to facilitate the calculation. The conversation highlights the importance of careful manipulation of the integrand to avoid errors in the convolution result. Ultimately, the correct approach is essential for accurately determining the convolution.
GuiltySparks
Messages
11
Reaction score
0

Homework Statement



Find the convolution of g(x) = e^{-πx^{2}} with itself from -∞ to ∞ using the definition of convolution, not the Fourier Transform.



The Attempt at a Solution



See my attachment. My professor said that you have to use integration by parts, but I keep getting stuck. No matter what I do, I reach the conclusion that the convolution is 0. Is there something that I'm missing here?
 

Attachments

  • gauss.jpg
    gauss.jpg
    11.8 KB · Views: 649
Physics news on Phys.org
GuiltySparks said:

Homework Statement



Find the convolution of g(x) = e^{-πx^{2}} with itself from -∞ to ∞ using the definition of convolution, not the Fourier Transform.

The Attempt at a Solution



See my attachment. My professor said that you have to use integration by parts, but I keep getting stuck. No matter what I do, I reach the conclusion that the convolution is 0. Is there something that I'm missing here?

Complete the square.

\displaystyle -2\pi\left( y^2-xy\right)-\pi x^2
\displaystyle =<br /> -2\pi\left( y^2-xy+\frac{x^2}{4}-\frac{x^2}{4}\right)-\pi x^2

\displaystyle =-2\pi\left( y-\frac{x}{2}\right)^2+2\pi\frac{x^2}{4}-\pi x^2

\displaystyle =-2\pi\left( y-\frac{x}{2}\right)^2-\pi\frac{x^2}{2}​
 
Question: A clock's minute hand has length 4 and its hour hand has length 3. What is the distance between the tips at the moment when it is increasing most rapidly?(Putnam Exam Question) Answer: Making assumption that both the hands moves at constant angular velocities, the answer is ## \sqrt{7} .## But don't you think this assumption is somewhat doubtful and wrong?

Similar threads

  • · Replies 10 ·
Replies
10
Views
2K
  • · Replies 1 ·
Replies
1
Views
1K
  • · Replies 19 ·
Replies
19
Views
29K
  • · Replies 3 ·
Replies
3
Views
1K
  • · Replies 14 ·
Replies
14
Views
2K
  • · Replies 1 ·
Replies
1
Views
3K
  • · Replies 11 ·
Replies
11
Views
7K
Replies
1
Views
1K
  • · Replies 5 ·
Replies
5
Views
2K
  • · Replies 1 ·
Replies
1
Views
1K